Difference between revisions of "2004 AMC 10A Problems/Problem 15"
(added category; fixed typo in answer choice) |
|||
Line 2: | Line 2: | ||
Given that <math>-4\leq x\leq-2</math> and <math>2\leq y\leq4</math>, what is the largest possible value of (x+y)/x? | Given that <math>-4\leq x\leq-2</math> and <math>2\leq y\leq4</math>, what is the largest possible value of (x+y)/x? | ||
− | <math> \mathrm{(A) \ } -1 \qquad \mathrm{(B) \ } -\frac12 \qquad \mathrm{(C) \ } 0 \qquad \mathrm{(D) \ } \frac12 | + | <math> \mathrm{(A) \ } -1 \qquad \mathrm{(B) \ } -\frac12 \qquad \mathrm{(C) \ } 0 \qquad \mathrm{(D) \ } \frac12 \qquad \mathrm{(E) \ } 1 </math> |
==Solution== | ==Solution== | ||
Line 20: | Line 20: | ||
*[[2004 AMC 10A Problems/Problem 16|Next Problem]] | *[[2004 AMC 10A Problems/Problem 16|Next Problem]] | ||
+ | |||
+ | [[Category:Introductory Algebra Problems]] |
Revision as of 11:12, 5 November 2006
Problem
Given that and , what is the largest possible value of (x+y)/x?
Solution
Rewrite as .
We also know that because and are of opposite parity.
Therefore, is maximized when is minimized, which occurs when is the largest and is the smallest.
This occurs at (-4,2), so .